Point A is a representation of which number on the number line?
A
B
D
-2
0
2
4
0 -3
0 1
T ♡rt

Answers

Answer 1
4 is the number of respresentation of point A ;)

Related Questions

what is the degree of the polynomial below

Answers

Answer:

d

Step-by-step explanation:

trust me gang

Answer:

the degree is the value of the highest exponent... 4x^3 the exponent is a three

answer "D"

Step-by-step explanation:

josh is standing on the top of a building that is 425 feet tall. He throws a penny up into the air with an initial of 32 ft/sec. How long does it take for the penny to hit the ground?
Y = -4.9x^2 + 32x + 425

A. 10 seconds

B. 6.25

C. 400

D. 0 seconds

Answers

Answer:

A

Step-by-step explanation:

cause it will be more faster just because it is in a solid phase or it is solid

The time taken to hit the ground is 6.25 sec.(Option B)

How to calculate time?

It is given that penny follows the trajectory:

[tex]y = -4.9x^2 + 32x + 425[/tex]

Differentiating it we get:

[tex]\dfrac{dy}{dx} =-9.8x+32[/tex]

At y=425, it is obvious that x=0

So dy/dx at x=0 is 32.

Now the vertical component of the velocity is 32sinФ (where Ф is the angle at which penny is thrown).

Ф[tex]=\tan^{-1}(32)[/tex]

Vertical component=[tex]32*\sin(\tan^{-1}(32))[/tex]=31.98 ft/sec

Now applying the equation of motion: acceleration=-32ft/sec^(2)

u=31.98ft/sec   s=-425   t=?

[tex]s=ut+\frac{1}{2} at^{2}[/tex]

[tex]-425=31.98t-16t^2[/tex]

[tex]16t^2-31.98t-425=0[/tex]

Solving the quadratic equation we get:

t=6.249 sec≅6.25 sec

Therefore, the time taken is 6.25 sec.

To know more about equation of motion refer:https://brainly.com/question/24966506

#SPJ2

For the function F defined by F(x)=x2−2x+4, find F(|−4|).
A. 8
B. 4
C. 12
D. 28

Answers

Answer:

12

Step-by-step explanation:

F(x)=x^2−2x+4

Let x = |-4| = 4

F(|-4|)=4^2−2(4)+4

     = 16 -8+4

    = 8+4

     = 12

I need help please. The question is already there. Tysm. 20 points.

Answers

Answer:

$109 and 35%

Step-by-step explanation:

49.05 is the sale price and the discount is 55%. What’s the original price?

price/ 1- discount

49.05/ 1 - 0.55

49.05/ 0.45

109

for the other part

to find the discount subtract final price from original price

94 - 61.10 = 32.9

Which would be 35% in percent form.

Here is a list of fractions 18/45 14/30 10/25 8/20 16/40 one of these fractions are not equivalent to 2/5 write down this fractions

Answers

Answer:

14/30

Step-by-step explanation:

How to simplify: • divide both numerator and denominator by their GCF.18/45= 18 ÷ 9 / 45 ÷ 9= 2/514/30= 14 ÷ 2 / 30 ÷ 2= 7/1510/25= 10 ÷ 5 / 25 ÷ 5= 2/58/20= 8 ÷ 4 / 20 ÷ 4= 2/516/40= 16 ÷ 8 / 40 ÷ 8= 2/5

[tex]\tt{ \green{P} \orange{s} \red{y} \blue{x} \pink{c} \purple{h} \green{i} e}[/tex]

[tex]\frac{14}{30}[/tex] is not equivalent to [tex]\frac{2}{5}[/tex] in the list of fractions [tex]\frac{18}{45}, \frac{14}{30} , \frac{x}{y} \frac{10}{25}, \frac{8}{20}, \frac{16}{40}[/tex].

Equivalent Fractions

Equivalent fractions represent the same value even though they look different.

How to determine the Equivalent fractions?

We know we can find an equivalent fraction of a given fraction by or dividing both the numerator and denominator of the given fraction with the same number (maybe LCM or HCF of the numerator or denominator).

[tex]\frac{18}{45}=\frac{18/9}{45/9}=\frac{2}{5}[/tex] (since [tex]9[/tex] is HCF of [tex]18, 45[/tex])

[tex]\frac{14}{30}=\frac{14/2}{30/2}=\frac{7}{15}[/tex] (since [tex]2[/tex] is HCF of [tex]7, 15[/tex])

[tex]\frac{10}{25} =\frac{10/5}{25/5} =\frac{2}{5}[/tex] (since [tex]5[/tex] is HCF of [tex]10, 25[/tex])

[tex]\frac{8}{20} =\frac{8/4}{20/4} =\frac{2}{5}[/tex] (since [tex]4[/tex] is HCF of [tex]8, 20[/tex])

[tex]\frac{16}{40} =\frac{16/8}{40/8} =\frac{2}{5}[/tex] (since [tex]8[/tex] is HCF of [tex]16, 40[/tex])

Thus, [tex]\frac{14}{30}[/tex] is not equivalent to [tex]\frac{2}{5}[/tex].

Learn more about Equivalent fractions here- https://brainly.com/question/17912

#SPJ2

Select the best answer to describe the parabola

Answers

Positive parabola that has a minimum.

It’s positive if it opens upward, negative if it opens downward. It has a minimum if it opens upward, and maximum when it opens downward.

Find the difference quotient for f(x) = x2 + 4.

Answers

I am trying to simplify a difference quotient with the form

f(x+h)−f(h)/h

if f(x)=2/x2 I have attempted to cancel out the denominator of the numerator by the least common denominator method. I know that I can solve this using implicit differentiation, but am trying to use the difference quotient to get the partial derivative

Here is my work so far:

f(x+h)−f(x)(h)

=f(2/(x+h)2)−f(2/x2)h

=(2/(x+h)2)−(2/x2)h

here I multiply by the LCD

=(2(x2)−2(x+h)2)/(x2(x+h)2)h

Here I expand out the top and cancel out 2x2

=(2x2−(2x2+4xh+2h2))/(x2(x+h)2)h

=−(4xh+2h2)/(x2(x+h)2))h

Here I think I should multiply by 1/h to get rid of the h on the bottom

I end up with:

=4xh+2h2hx2(x+h)2

factor out an h

=−4x+2hx2(x+h)2

Help me! thank you so much

Answers

Answer:

Step-by-step explanation:

[tex]\frac{sinxcos^3x-cos xsin^3x}{cos^42x-sin^42x} \\=\frac{sin x cos x(cos^2x-sin ^2 x)}{(cis^2 2x+sin^2 2x)(cos^2 2x-sin ^22x)} \\=\frac{2sin x cos x cos 2x}{2(1)(cos 4x)} \\=\frac{sin 2x cos 2x}{2 cos 4x} \\=\frac{2 sin 2x cos 2x}{4 cos 4x} \\=\frac{sin 4x}{4 cos 4x} \\=\frac{1}{4} tan 4x[/tex]

help lol i forgot everything of the summer time

fill in the table using this function rule

Answers

Answer:

hope it help you

Step-by-step explanation:

mark me brailiest answer

What I do is plug x into the equation to find y

1. Y=-4(-1)+3
Y=4+3
Y=7
2. Y=-4(0)+3
Y=0+3
Y=3
3. Y=-4(1)+3
Y=-4+3
Y=-1
4. Y=-4(2)+3
Y=-8+3
Y=-5

Hope this helps!! :)

4. a) x2 + xy2 + y 4

Answers

Answer:

x2 + xy2 + y 4=(x+y^2)^2-xy^2

Step-by-step explanation:

Helpppp pleaseeee !!!!!!

Answers

Answer:

149 inches squared

Step-by-step explanation:

top rectangle: 25 * 7 = 175

second rectangle: 8 * (25 - 17) = 8^2 = 64

triangle in bottom right: 1/2 * (13 - 8) * (15 - 11) = 10

175 + 64 + 10 = 149 sq in

hopefully got this right!

find the missing side. Round it the nearest tenth.​

Answers

Answer: x= 11√3= 19.0525 = 19.1

Step-by-step explanation:

Let the reference angle be 30

so

cos 30 = b/h

√3/2 = x/22

or, 22√3 = 2x

or. x = (22√3)/2

so, x = 11√3

Answer:

x = 19.1 cm

Step-by-step explanation:

→ Find the name of the side you are not given

Opposite

→ Find a formula without opposite in it

Cos = Adjacent ÷ Hypotenuse

→ Rearrange to make adjacent the subject

Adjacent = Cos × Hypotenuse

→ Substitute in the values

Adjacent = Cos ( 30 ) × 22

→ Simplify

Adjacent = 19.1

If give 7 billions for 7 millions people. What is total?

Answers

Answer:

this is a very big number

Step-by-step explanation:

(7000000000)⁷⁰⁰⁰⁰⁰⁰

Evaluate 4(3 - 1)^2..

Answers

Answer:

16

Step-by-step explanation:

4(3 - 1)^2

~Simplify using PEMDAS

4(2)^2

4(4)

16

Best of Luck!

Difference of the square of (x+6)(x+4)

Answers

Answer:

x^2 + 10x + 24

Step-by-step explanation:

(x+6)(x+4)

x^2 + 4x + 6x + 24

x^2 + 10x + 24

Amy, a nature photographer, randomly sampled photographs she took within the last year. She wanted to find out how many of her photographs contained flowers. The proportion of photographs that had flowers was 0.61, with a margin of error of 0.04. Construct a confidence interval for the proportion of her photographs taken within the last year contained flowers.

Answers

The Constructed  confidence interval for the proportion of her photographs taken within the last year contained flowers is

[tex]CI\ E(0.57,0.65)[/tex]

From the question we are told that:

The proportion of photographs that had flowers P= 0.61

Margin of error of M.E= 0.04

Generally, the equation for Confidence interval for proportion is mathematically given by

[tex]CI=0.61 \pm 0.04[/tex]P \pm M.E

Therefore Confidence interval is

[tex]CI=0.61 \pm 0.04[/tex]

And can also be written as

[tex]CI\ E((0.61+0.04),0.61-0.04))[/tex]

[tex]CI\ E(0.57,0.65)[/tex]

In conclusion the Confidence interval is

[tex]CI\ E(0.57,0.65)[/tex]

For more information on this visit

https://brainly.com/question/24131141?referrer=searchResults

7x+2y=19
7x+8y=13
Please solve it by Simultaneous method

Answers

Answer:

y=-1

x=3

Step-by-step explanation:

7x+2y=19-----(1)

7x+8y=13------(2)

(1)-(2)

(7x+2y)-(7x+8y)=19-13

7x+2y-7x-8y=6

+2y-8y=6

-6y=6

y=-1

by substitute in (1)

7x+2*(-1)=19

7x-2=19

7x=19+2

7x=21

x=21/3

x=3

Jay has a jar of beads. 6 purple beard, 8 blue beads, and 4 yellow beads. If he removes one bead at random, what is the probability that it will not be yellow?

Answers

You get a pie chart to do percentages just so you dont cheat you still can do work y'know

What would it be tho 300 doesn’t show up in my options my options are
1/49 -1/49 -49 and 49

Answers

1/49 would be the answer

SAT/ACT What is the solution of 1,200 – 5(3x + 30) = 600? A 30 B 50 C 150 D 200 E 250​

Answers

The answer you are looking for is letter A, x=30.

Solution/Explanation:

First, write out the equation,

1200-5(3x+30)=600

Next, using the Distributive Property,

1200-15x-150=600

Simplify the left side of the equation just a little bit more,

1050-15x=600

Reverse order of terms on the left side, to make it a little bit easier to solve,

-15x+1050=600

Now, subtract 1050 from both sides,

-15x+1050-1050=600-1050

Now, simplify this part of the equation,

-15x=-450

Finally, divide both sides by -15,

So, therefore, the final answer is x=30.

I hope this helped you. Enjoy your day, and take care!

I NEEDDD HELPPP ITSSSSSS URGENTTTTT!!!

Answers

Answer:

100 degrees

Step-by-step explanation:

measure of FHD = 65 + 35

measure of FHD = 100

Basically count/add up the total amount of degrees that are include in the angle <FHD.

-- (central angles)

So, 35 + 65 = 100 degrees

Find the length of x

Answers

Answer:

20

Step-by-step explanation:

Since the triangles are similar then the sides must be proportional

10/6 = x/12 cross multiply expressions

6x = 120 divide both sides by 6

x = 20

The system of linear equations x – y + 2 = 0 and 2x + y -14 = 0​

Answers

Answer:

(4, 6 )

Step-by-step explanation:

Given the equations

x - y + 2 = 0 ( subtract - y + 2 from both sides )

x = y - 2 → (1)

2x + y - 14 = 0 → (2)

Substitute x = y - 2 into (2)

2(y - 2) + y - 14 = 0 ← distribute and simplify left side

2y - 4 + y - 14 = 0

3y - 18 = 0 ( add 18 to both sides )

3y = 18 ( divide both sides by 3 )

y = 6

Substitute y = 6 into (1) for corresponding value of x

x = 6 - 2 = 4

solution is (4, 6 )

Given 12 consecutive integers, how many ways can three of these integers be selected to give a sum which divides by 4.

Disclaimer: A lot of points to be given, Full explanation required. Not only answer. Remember the sum of the number must be divisible by 4. I think modular arithmetic is the way to solve it, but who knows???

Answers

Answer:

55 ways

Step-by-step explanation:

Out of 12 consecutive integers:

3 - divide by 4, so the remainder is 0 3- give remainder of 1 3- give remainder of 2 3 - give remainder of 3

Sum of 3 integers will be divisible by 4 if the remainders are:

0 - 0 - 0 ⇒ 1 combination 0 - 1 - 3 ⇒ 3*3 = 9 combinations 0 - 3 - 1 ⇒ 3*3 = 9  combinations 1 - 1 - 2  ⇒ 2*3 = 6  combinations 1 - 2 - 1  ⇒ 2*3 = 6  combinations 2 - 1 - 1 ⇒ 2*3 = 6 combinations 3 - 0 - 1 ⇒ 3*3 = 9 combinations 3 - 1 - 0 ⇒ 3*3 = 9 combinations

So total number of combinations is:

1 + 9*4 + 6*3 = 55

Lilian is building a swimming pool in the shape of a right rectangular prism. The area of the base of the swimming pool is 72 square meters. The depth of the swimming pool is 3 meters. What is the volume of the swimming pool?

Answers

Answer:

216

Step-by-step explanation:

Volume of a rectangular prism = area of base * depth

Area of base: 72

Depth: 3

Volume = 72 * 3 = 216

What is an example in your house of a point ?

Answers

Answer:

Step-by-step explanation:

Nothing. A point is dimensionless. Ugh what an awful LOL.

How about the tip of a candle that has never been lit?

What is the smallest 3-digit palindrome that is divisible by both 3 and 4?

Answers

Answer:

252

Step-by-step explanation:

To be divisible by 3, it's digits have to add to a number that is a multiple of 3.

To be divisible by 4 its last 2 digits have to be divisible by 3.

So let's start with 1x1 which won't work because 1x1 is odd. so let's go to 2x2 and see what happens.

212 that's divisible by 4 but not 3

222 divisible by 3 but not 4

232 divisible by 4 but not 3

242 not divisible by either one.

252 I think this might be your answer

The digits add up to 9 which is a multiple of 3 and the last 2 digits are divisible by 4

2. Determine the measure of the angles indicated by letters. Justify your answers with the
properties or theorems you used.

Answers

Answer:

a = 50°

b = 130°

c = 50°

d = 50°

e = 130°

f = 130°

g = 50°

Answered by GAUTHMATH

3,125 subtracted by what can give me 514

Answers

Answer:

2611

Step-by-step explanation:

3125-2611= 514

for more answers check my bio

PLS HELP ME ON THIS QUESTION I WILL MRK YOU AS BRAINLIEST IF YOU KNOW THE ANSWER!!
Which of the following measures is a measure of spread?
A. median
B. range
C. mode
D. mean

Answers

Answer:

range

Step-by-step explanation:

Answer:

B. range.

Step-by-step explanation:

others are:

» Standard variation.

» Interquatile range.

» Quatiles, deciles and percentiles.

» variance.

[tex]{ \underline{ \blue{ \sf{christ \: † \: alone}}}}[/tex]

Other Questions
A rapidly expanding percentage of urban residents live in:high-rise buildingsslumssingle-family homespublic housing Mr. Ziegler is turning 65 next month and has asked you what he can do, and when he must do it, with respect to enrolling in Part D. What could you tell him? Algebra 1 need help ASAP Four people share a taxi to the airport the fare was $36 and they gave the driver a tip equal to 25% of the fair. If they equally share the cost of the fair tip, how How much did each person pay? A binomial with a leading coefficient of 4 and a constant of 1. 2/5 = 6/ = 10/30 = 14/ Please tell asap 4. When setting goals, you should do everything EXCEPT which of the following?Take into account your current level of activity.Ask all of your friends what they think your goals should be..Set a time limit.Be realistic. ASAPIdentify the error in the following statement: Photosynthesis is a chemical reaction that takes place in the chloroplasts of green plant cells, where light energy is used to convert glucose and water into carbon dioxide and oxygen.A. glucose and carbon dioxide should be switchedB. No errorsC. water and oxygen should be switchedD. chloroplasts should be mitochondria where and in which condition gold is found? Based on previous knowledge of Greek values, what prediction can best be made about Antinous's future? Antinous's friends will rally around him, and he will be victorious in battle. Antinous will gain great power and win the heart of Penelope at last. Antinous's friends will turn on him, and he will suffer for his actions. Antinous will become a poor man and wanderer like Odysseus. Rs 600 will gave Rs 90 simple interest at 5 % per annum. why cells are sometimes called the building block of life? A storeowner orders 25 calculators that cost $38 each. The storeowner can sell each calculator for $42. The storeowner sold 22 calculators to customers. He had to return 3 calculators that were never sold and pay a $2 charge for each returned calculator (although the initial cost is refunded). What is the storeowner's profit? A square steel bar has a length of 5.1 ft and a 2.7 in by 2.7 in cross section and is subjected to axial tension. The final length is 5.10295 ft . The final side length is 2.69953 in . What is Poisson's ratio for the material Who was Theodore Roosevelt and how was he affected by succession? Midsouth Stitchery wants to improve their productivity. Their process yield is currently 91.56% based on 9816 yards of material. If they are going to improve their productivity to 92.10, how many yards of finished material will they have to produce from the same amount of material input Select the correct answer.Who can propose an amendment to the Constitution?A a simple majority of CongressB. the president.the Supreme CourtD.a two-thirds majority of Congress Question in the picture please help me...Show all steps please.... Please help! Thank you!!!!! solve the inequality y-6>/2y-4